Simplifying natural log of complex number

Click For Summary
The discussion focuses on simplifying the natural logarithm of a complex function, specifically Log[(z+1)/(z-1)]. Participants explore how to express the function in the form u + iv by breaking it down into its components. The approach involves using properties of logarithms, such as separating the logarithm of a quotient into the difference of two logarithms. Key calculations include determining the modulus and argument of the complex numbers involved, leading to the realization that ln(z+1) can be expressed as ln[(x+1)^2 + y^2]^(1/2) + i[arctan(y/(x+1))]. The conversation concludes with participants gaining clarity on the generalization of logarithmic properties for complex numbers.
Sturk200
Messages
168
Reaction score
17

Homework Statement


The problem is to sketch lines of constant u and v in the image plane for the function Log[(z+1)/(z-1)].

Homework Equations



z=x+iy

The Attempt at a Solution



In order to do this I have to get the expression into u+iv form, so that I can read off and manipulate the u and v aspects of the function. What I can't figure out is how to get this equation into u+iv form.

I know that for Ln[z], I can write Ln[reitheta] = Ln[r] + itheta. Then u=Ln[r] and v=theta. But having that added +1 in the natural log is really throwing me off. Does anyone have any ideas for how to get this into u+iv form? Is there some obvious rule for simplifying the logarithm of a sum that I'm forgetting?

Thanks.
 
Physics news on Phys.org
\ln f(z) = \ln |f(z)| + i\arg(f(z)).

It might be easier to write \ln\frac{z + 1}{z-1} = \ln(z+1) - \ln(z - 1) first.
 
pasmith said:
\ln f(z) = \ln |f(z)| + i\arg(f(z)).

It might be easier to write \ln\frac{z + 1}{z-1} = \ln(z+1) - \ln(z - 1) first.

Thanks for your response. I think I'm starting to get it now that I've seen it in general form. Here is what I'm getting:

The modulus of (z+1) is [(x+1)2+y2]1/2, since 1 is a real number and therefore addends itself to the x-component of the modulus.

The argument of z+1 remains unchanged and is equal to theta.

Therefore ln(z+1) = ln[(x+1)^2 + y^2]^1/2 + i(theta).

Is that on the right track?

I'm also wondering how you go about justifying the generalization in the first line of your post. It's clearly true when f(z)=z, but how do you get from there to the general case of taking the modulus and the argument more abstractly?
 
Sturk200 said:
Thanks for your response. I think I'm starting to get it now that I've seen it in general form. Here is what I'm getting:

The modulus of (z+1) is [(x+1)2+y2]1/2, since 1 is a real number and therefore addends itself to the x-component of the modulus.

The argument of z+1 remains unchanged and is equal to theta.

Are you sure? \tan (\arg(z+1)) = \frac{y}{x+1} which is not in general equal to \frac yx.

Therefore ln(z+1) = ln[(x+1)^2 + y^2]^1/2 + i(theta).

Is that on the right track?

I'm also wondering how you go about justifying the generalization in the first line of your post. It's clearly true when f(z)=z, but how do you get from there to the general case of taking the modulus and the argument more abstractly?

z in \ln z = \ln |z| + i\arg(z) is just a (non-zero) complex number. You can replace it with any other collection of symbols which represent a (non-zero) complex number and the equation still holds.
 
pasmith said:
Are you sure? tan(arg(z+1))=yx+1\tan (\arg(z+1)) = \frac{y}{x+1} which is not in general equal to yx\frac yx.

O boy, that was pretty dumb. I think I'm actually getting it now.

So ln(z+1) = ln[(x+1)^2+y^2] 1/2+ i[arctan(y/(x+1))]. Is that right?

pasmith said:
zz in lnz=ln|z|+iarg(z)\ln z = \ln |z| + i\arg(z) is just a (non-zero) complex number. You can replace it with any other collection of symbols which represent a (non-zero) complex number and the equation still holds.

That makes so much sense, I don't know why I had a block on it before. Thanks again!
 
Question: A clock's minute hand has length 4 and its hour hand has length 3. What is the distance between the tips at the moment when it is increasing most rapidly?(Putnam Exam Question) Answer: Making assumption that both the hands moves at constant angular velocities, the answer is ## \sqrt{7} .## But don't you think this assumption is somewhat doubtful and wrong?

Similar threads

  • · Replies 27 ·
Replies
27
Views
2K
  • · Replies 5 ·
Replies
5
Views
2K
  • · Replies 13 ·
Replies
13
Views
3K
  • · Replies 3 ·
Replies
3
Views
2K
  • · Replies 17 ·
Replies
17
Views
2K
  • · Replies 3 ·
Replies
3
Views
2K
  • · Replies 4 ·
Replies
4
Views
3K
  • · Replies 17 ·
Replies
17
Views
3K
  • · Replies 6 ·
Replies
6
Views
2K
  • · Replies 3 ·
Replies
3
Views
3K